LSAT and Law School Admissions Forum

Get expert LSAT preparation and law school admissions advice from PowerScore Test Preparation.

 Administrator
PowerScore Staff
  • PowerScore Staff
  • Posts: 8916
  • Joined: Feb 02, 2011
|
#36944
Complete Question Explanation

Weaken. The correct answer choice is (D)

Barr’s argument, as presented in the stimulus, is as follows: Based on the premise that Survey Group has
tracked tea sales at numerous stores over the last 20 years and observed no increase in tea sales, it can be
concluded that tea’s popularity has not actually increased (in spite of the recent increase in visibility).

Often an LSAT argument will rest on evidence provided by a survey, but we should be wary of
conclusions drawn from polls which are potentially unreliable. In this case, Barr’s argument is weak,
because Survey Group might be using a non-representative sample. “Numerous” is vague, and does not
guarantee that Survey Group has polled a number sufficient to produce a good sample, so the survey
results may be unreliable. Since we are asked to weaken Barr’s argument, we should watch for an
answer choice which reflects this flaw.

Answer choice (A): This choice provides no actual evidence, only amorphous plans for a future survey
to be conducted by a special interest group. Since this answer has no effect on the strength of the
argument in the stimulus, it certainly doesn’t weaken the conclusion and is thus incorrect.

Answer choice (B): This choice does not weaken the argument that tea’s popularity has not actually
increased. It might establish only that amongst people who drink tea, there is receptiveness to variety.

Answer choice (C): This choice concerns only the source of the evidence discussed, but information
about funding does not provide evidence of a particular survey’s validity. In any case, it is not possible to
know whether “consumer advocacy groups” would make the study more accurate, less accurate, or have
any effect at all.

Answer choice (D): This is the correct answer choice. If the stores are all located in the same small
region, there is a good chance that the sample is not representative, and that tea-sales could have vastly
increased in other, unstudied, areas of the country. This response effectively attacks the statistical flaw in
Barr’s argument.

Answer choice (E): Even if a campaign is expensive and efficient, this does not prove it successful, so
this response does not weaken the argument.

Get the most out of your LSAT Prep Plus subscription.

Analyze and track your performance with our Testing and Analytics Package.